Difference between revisions of "User:Nik"

From Ferienserie MMP2
Jump to: navigation, search
(Added an alternative solution)
 
(45 intermediate revisions by 2 users not shown)
Line 1: Line 1:
== Foreword ==
+
Niklaus Messerli
I use \(Q\:/\:P\) for the transformed system instead of \(\widetilde{q}\:/\:\widetilde{p}\) because it's easier to write in Latex.  
+
nik@student.ethz.ch
== Problem ==
+
Let
+
\( \Phi \in C^\infty(\mathbb{R}^{2n}) \)
+
have the property that the system
+
\( p_i = \frac{\partial}{\partial q_i} \Phi (q, Q) \)
+
has a unique smooth solution
+
\( Q = Q(q,p) \).  
+
  
Define
+
{| class="wikitable"
\( P_i(q,p) = - \frac{\partial}{\partial Q_i} \Phi (q, Q) | _{Q= Q(q,p)} \)
+
!Article
 
+
!Check
Let \( \{\cdot,\cdot\} \) be the Poisson bracket, such that
+
|-
\(  \{f,g\} = \sum_{j=1}^n \frac{\partial f}{\partial q_j} \frac{\partial g}{\partial p_j} - \frac{\partial f}{\partial p_j} \frac{\partial g}{\partial q_j} \)
+
| [[Aufgaben:Problem 1|Problem 1]]
 
+
| 1
Show that:
+
|-
 
+
| [[Aufgaben:Problem 2|Problem 2]]
I) \( \{Q_i(q,p), Q_j(q,p)\} = 0 \)
+
| a-c):1, d):0
 
+
|-
II) \( \{Q_i(q,p), P_j(q,p)\} = \delta_{ij} \)
+
| [[Aufgaben:Problem 3|Problem 3]]
 
+
| d) Wieso müssen wir zeigen dass "\(\{L_g: g \in G\}\) is a group under composition"? Wir wissen doch schon, dass \(L: G \rightarrow \mathrm{Sym}G\) und dass \(G\) und \(\mathrm{Sym}G\) Gruppen sind.
III) \( \{P_i (q,p), P_j(q,p)\} = 0 \)
+
|-
 
+
| [[Aufgaben:Problem 4|Problem 4]]
 
+
| 1
 
+
|-
==Solution==
+
| [[Aufgaben:Problem 5|Problem 5]]
 
+
| 1
===Important equations===
+
|-
 
+
| [[Aufgaben:Problem 6|Problem 6]]
$$ \underbrace{\frac{\partial p_i}{\partial q_k}}_{0} = \frac{\partial}{\partial q_k}\left(\frac{\partial \Phi}{\partial q_i}(q, Q)\right) $$
+
| 1
$$ = \sum_{s=1}^{n}\left(\frac{\partial^2 \Phi}{\partial q_i \partial q_s}(q, Q)\underbrace{\frac{\partial q_s}{\partial q_k}}_{\delta_{sk}} + \frac{\partial^2 \Phi}{\partial q_i \partial Q_s}(q, Q)\frac{\partial Q_s}{\partial q_k}\right) $$
+
|-
$$ \Rightarrow 0 = \frac{\partial^2 \Phi}{\partial q_i \partial q_k}(q, Q) + \sum_{s=1}^{n}\left(\frac{\partial^2 \Phi}{\partial q_i \partial Q_s}(q, Q)\frac{\partial Q_s}{\partial q_k}\right) $$
+
| [[Aufgaben:Problem 7|Problem 7]]
$$ \tag{1} \Rightarrow \boxed{ -\frac{\partial^2 \Phi}{\partial q_i \partial q_k}(q, Q) = \sum_{s=1}^{n}\left(\frac{\partial^2 \Phi}{\partial q_i \partial Q_s}(q, Q)\frac{\partial Q_s}{\partial q_k}\right) } $$
+
| 1
 
+
|-
 
+
| [[Aufgaben:Problem 8|Problem 8]]
$$ \underbrace{\frac{\partial p_i}{\partial p_k}}_{\delta_{ik}} = \frac{\partial}{\partial p_k}\left(\frac{\partial \Phi}{\partial q_i}(q, Q)\right) $$
+
| 2
$$ = \sum_{s=1}^{n}\left(\frac{\partial^2 \Phi}{\partial q_i \partial q_s}(q, Q)\underbrace{\frac{\partial q_s}{\partial p_k}}_{0} + \frac{\partial^2 \Phi}{\partial q_i \partial Q_s}(q, Q)\frac{\partial Q_s}{\partial p_k}\right) $$
+
|-
$$ \tag{2} \boxed{ \Rightarrow \delta_{ik} = \sum_{s=1}^{n}\left(\frac{\partial^2 \Phi}{\partial q_i \partial Q_s}(q, Q)\frac{\partial Q_s}{\partial p_k}\right) } $$
+
| [[Aufgaben:Problem 9|Problem 9]]
 
+
| 1
 
+
|-
$$ \frac{\partial P_i}{\partial q_k} = \frac{\partial}{\partial q_k}\left(-\frac{\partial \Phi}{\partial Q_i}(q, Q)\right) $$
+
| [[Aufgaben:Problem 10|Problem 10]]
$$ = \sum_{s=1}^{n}\left(-\frac{\partial^2 \Phi}{\partial Q_i \partial q_s}(q, Q)\underbrace{\frac{\partial q_s}{\partial q_k}}_{\delta_{sk}} - \frac{\partial^2 \Phi}{\partial Q_i \partial Q_s}(q, Q)\frac{\partial Q_s}{\partial q_k}\right) $$
+
| a by craven seems legit
$$ \tag{3} \boxed{ \Rightarrow \frac{\partial P_i}{\partial q_k} = -\frac{\partial^2 \Phi}{\partial Q_i \partial q_k}(q, Q) - \sum_{s=1}^{n}\left(\frac{\partial^2 \Phi}{\partial Q_i \partial Q_s}(q, Q)\frac{\partial Q_s}{\partial q_k}\right) } $$
+
|-
 
+
| [[Aufgaben:Problem 11|Problem 11]]
 
+
| 1
$$ \frac{\partial P_i}{\partial p_k} = \frac{\partial}{\partial p_k}\left(-\frac{\partial \Phi}{\partial Q_i}(q, Q)\right) $$
+
|-
$$ = \sum_{s=1}^{n}\left(-\frac{\partial^2 \Phi}{\partial Q_i \partial q_s}(q, Q)\underbrace{\frac{\partial q_s}{\partial p_k}}_{0} - \frac{\partial^2 \Phi}{\partial Q_i \partial Q_s}(q, Q)\frac{\partial Q_s}{\partial p_k}\right) $$
+
| [[Aufgaben:Problem 12|Problem 12]]
$$ \tag{4} \Rightarrow \boxed{ \frac{\partial P_i}{\partial p_k} = -\sum_{s=1}^{n}\left(\frac{\partial^2 \Phi}{\partial Q_i \partial Q_s}(q, Q)\frac{\partial Q_s}{\partial p_k}\right) } $$
+
| bitch please
 
+
|-
 
+
| [[Aufgaben:Problem 13|Problem 13]]
===Solution I)===
+
|
This is the most complicated problem but we calculate it first because we use it in II) und III)
+
|-
First we write equations (1) and (2) again an shift our eyeballs between them and look at it as matrix multiplication:  
+
| [[Aufgaben:Problem 14|Problem 14]]
$$ \underbrace{-\frac{\partial^2 \Phi}{\partial q_i \partial q_k}(q, Q)}_{D_{ik}} = \sum_{s=1}^{n}\left(\underbrace{\frac{\partial^2 \Phi}{\partial q_i \partial Q_s}(q, Q)}_{A_{is}}\underbrace{\frac{\partial Q_s}{\partial q_k}}_{C_{sk}}\right) $$
+
|
$$ \underbrace{\delta_{ik}}_{\mathbb{1}} = \sum_{s=1}^{n}\left(\underbrace{\frac{\partial^2 \Phi}{\partial q_i \partial Q_s}(q, Q)}_{A_{is}}\underbrace{\frac{\partial Q_s}{\partial p_k}}_{B_{sk}}\right) $$
+
|-
Now let's do some linear algebra: We know that \( \mathbb{1} = AB \Rightarrow A^{-1} = B \text{ and } D = AC \)
+
| [[Aufgaben:Problem 15|Problem 15]]
$$ \Rightarrow A^{-1}D = C $$
+
| a): 1
$$\Rightarrow BD = C $$
+
|-
With that we can now calculate C which we need to calculate the Poisson bracket
+
|}
$$ C_{sk} = - \sum_{e=1}^{n}\left(\frac{\partial Q_s}{\partial p_e}\frac{\partial^2 \Phi}{\partial q_k \partial q_e}\right) $$
+
Now we change the indices such that it fits into the equation
+
$$ \frac{\partial Q_i}{\partial q_k} = - \sum_{s=1}^{n}\left(\frac{\partial Q_i}{\partial p_s}\frac{\partial^2 \Phi}{\partial q_k \partial q_s}\right) $$
+
Finally we are prepared to calculate the Poisson bracket
+
$$ \{Q_i, Q_j\} = \sum_{k=1}^{n}\left(\frac{\partial Q_i}{\partial q_k}\frac{\partial Q_j}{\partial p_k} - \frac{\partial Q_i}{\partial p_k}\frac{\partial Q_j}{\partial q_k}\right) $$
+
Put in the sum from above
+
$$ = \sum_{k=1}^{n}\left(\frac{\partial Q_j}{\partial p_k}\left(-\sum_{s=1}^{n}\left(\frac{\partial Q_i}{\partial p_s}\frac{\partial^2 \Phi}{\partial q_k \partial q_s}(q, Q)\right)\right) - \frac{\partial Q_i}{\partial p_k}\left(-\sum_{s=1}^{n}\left(\frac{\partial Q_j}{\partial p_s}\frac{\partial^2 \Phi}{\partial q_k \partial q_s}(q, Q)\right)\right)\right) $$
+
Take the sum out
+
$$ \sum_{k=1}^{n}\sum_{s=1}^{n}\left(\frac{\partial Q_i}{\partial p_k}\frac{\partial Q_j}{\partial p_s}\frac{\partial^2 \Phi}{\partial q_k \partial q_s}(q, Q) - \frac{\partial Q_j}{\partial p_k}\frac{\partial Q_i}{\partial p_s}\frac{\partial^2 \Phi}{\partial q_k \partial q_s}(q, Q) \right) = 0 $$
+
Since we sum with \(s\) and \(k\) completely from \(1\) to \(n\) the two parts are equal and therefore the difference is zero.
+
 
+
===Solution II)===
+
 
+
$$ \{Q_i, P_j\} = \sum_{k=1}^{n}\left( \frac{\partial Q_i}{\partial q_k}\frac{\partial P_j}{\partial p_k} - \frac{\partial Q_i}{\partial p_k}\frac{\partial P_j}{\partial q_k} \right) $$
+
Now we put in the results from equation (3) and (4)
+
$$ = \sum_{k=1}^{n}\left(\frac{\partial Q_i}{\partial q_k}\left(-\sum_{s=1}^{n}\left(\frac{\partial^2 \Phi}{\partial Q_j \partial Q_s}(q, Q)\frac{\partial Q_s}{\partial p_k}\right)\right) - \frac{\partial Q_i}{\partial p_k}\left(-\frac{\partial^2 \Phi}{\partial Q_j \partial q_k}(q, Q) - \sum_{s=1}^{n}\left(\frac{\partial^2 \Phi}{\partial Q_j \partial Q_s}(q, Q)\frac{\partial Q_s}{\partial q_k}\right)\right)\right) $$
+
Now we multiply out and separate all the sums. We can rearrange it like this because all the parts we put inside are independent on the running variable.  
+
$$ =  \sum_{k=1}^{n}\left(\frac{\partial Q_i}{\partial p_k}\frac{\partial^2 \Phi}{\partial Q_j \partial q_k}(q, Q)\right) + \sum_{k=1}^{n}\sum_{s=1}^{n}\left(\frac{\partial Q_i}{\partial p_k}\frac{\partial^2 \Phi}{\partial Q_j \partial Q_s}(q, Q)\frac{\partial Q_s}{\partial q_k}\right) - \sum_{k=1}^{n}\sum_{s=1}^{n}\left(\frac{\partial Q_i}{\partial q_k}\frac{\partial^2 \Phi}{\partial Q_j \partial Q_s}(q, Q)\frac{\partial Q_s}{\partial p_k}\right) $$
+
Now we rearrange again such that we can build a new Poisson bracket
+
$$ =  \underbrace{\sum_{k=1}^{n}\left(\frac{\partial Q_i}{\partial p_k}\frac{\partial^2 \Phi}{\partial Q_j \partial q_k}(q, Q)\right)}_{\color{red}{\delta_{ij}}} + \sum_{s=1}^{n}\left(\frac{\partial^2 \Phi}{\partial Q_j \partial Q_s}(q, Q) \underbrace{\sum_{k=1}^{n}\left(\frac{\partial Q_i}{\partial p_k}\frac{\partial Q_s}{\partial q_k} - \frac{\partial Q_i}{\partial q_k}\frac{\partial Q_s}{\partial p_k}\right)}_{=\{Q_s, Q_i\}=0}\right) $$
+
Now we use the definition from the problem and then the chain rule
+
$$ =  \sum_{k=1}^{n}\left(\frac{\partial Q_i}{\partial p_k}\frac{\partial p_k}{\partial Q_j} \right) = \frac{\partial Q_i}{\partial Q_j} = \delta_{ij} $$
+
<span style="color:red">The chain rule is here not correct. It would imply that \( Q \) is independent of \( q \) and that is not correct. Since I got no better solution I would write \( \delta_{ij} \) directly in the sum above. </span>
+
 
+
===Solution III)===
+
 
+
$$ \{P_i, P_j\} = \sum_{k=1}^{n}\left(\frac{\partial P_i}{\partial q_k}\frac{\partial P_j}{\partial p_k} - \frac{\partial P_i}{\partial p_k}\frac{\partial P_j}{\partial q_k}\right) $$
+
Now we put in the results from equation (3) and (4) in all 4 parts of the sum
+
$$ = \sum_{k=1}^{n}\left(\left(-\frac{\partial^2 \Phi}{\partial Q_i \partial q_k}(q, Q) - \underbrace{\sum_{s=1}^{n}\left(\frac{\partial^2 \Phi}{\partial Q_i \partial Q_s}(q, Q)\frac{\partial Q_s}{\partial q_k}\right)}_{(a)}\right)\left(\underbrace{-\sum_{s=1}^{n}\left(\frac{\partial^2 \Phi}{\partial Q_j \partial Q_s}(q, Q)\frac{\partial Q_s}{\partial p_k}\right)}_{(b)}\right) \\ - \left(\underbrace{-\sum_{s=1}^{n}\left(\frac{\partial^2 \Phi}{\partial Q_i \partial Q_s}(q, Q)\frac{\partial Q_s}{\partial p_k}\right)}_{(c)}\right)\left(-\frac{\partial^2 \Phi}{\partial Q_j \partial q_k}(q, Q) - \underbrace{\sum_{s=1}^{n}\left(\frac{\partial^2 \Phi}{\partial Q_j \partial Q_s}(q, Q)\frac{\partial Q_s}{\partial q_k}\right)}_{(d)}\right)\right) $$
+
If we multiply out we see that (a) * (b) = (c) * (d) and because of the sign they cancel out.
+
So we multiply out the remaining parts
+
$$ = \sum_{k=1}^{n}\sum_{s=1}^{n}\left(\frac{\partial^2 \Phi}{\partial Q_i \partial q_k}(q, Q)\frac{\partial^2 \Phi}{\partial Q_j \partial Q_s}(q, Q)\frac{\partial Q_s}{\partial p_k} - \frac{\partial^2 \Phi}{\partial Q_i \partial Q_s}(q, Q)\frac{\partial Q_s}{\partial p_k}\frac{\partial^2 \Phi}{\partial Q_j \partial q_k}(q, Q)\right) $$
+
And use the definition from the problem set
+
$$ = \sum_{k=1}^{n}\sum_{s=1}^{n}\left(-\frac{\partial^2 \Phi}{\partial Q_i \partial q_k}(q, Q)\underbrace{\frac{\partial P_s}{\partial Q_j}}_{=0}\frac{\partial Q_s}{\partial p_k} + \underbrace{\frac{\partial P_s}{\partial Q_i}}_{=0}\frac{\partial Q_s}{\partial p_k}\frac{\partial^2 \Phi}{\partial Q_j \partial q_k}(q, Q)\right) $$
+
 
+
 
+
 
+
$$ = 0 $$
+
 
+
 
+
===Alternative Solution===
+
 
+
'''Alternative solution:'''
+
 
+
Here's a proof I've found on my usual source of knowledge (meaning a youtube-lecture):
+
 
+
'''Claim:''' For a generating function of type one (that's the smartman's name for the Phi in our problem) it holds: \( \{ Q, P \} = 1 \).
+
 
+
''Proof:''
+
 
+
We first fix the \( q \) coordinates in the momentum equation and derive by \( p \), denoted: \( \Phi_p |_q\).
+
 
+
So we get:
+
 
+
'''Equation 1:'''
+
 
+
$$ 1 = \Phi_{q Q} Q_p |_q \Rightarrow Q_p |_q = \frac{1}{\Phi_{q Q}} $$
+
 
+
'''Equation 2:'''
+
 
+
Holding \( p \) fixed and derive by \( q \):  
+
 
+
$$ 0 = \frac{\partial p}{\partial q} = \Phi_{qq} + \Phi_{qQ}Q_{q}|_p \Rightarrow Q_{q}|_p = - \frac{\Phi_{qq}}{\Phi_{qQ}} $$
+
 
+
'''Equation 3:'''
+
 
+
Holding \( q \) fixed and derive by \( p \):
+
 
+
$$ P_p |_q = - \Phi_{QQ}Q_q |_p \Rightarrow P_p |_q = - \frac{\Phi_{QQ}}{\Phi_{qQ}} $$
+
 
+
where we used equation 1 again.
+
 
+
'''Equation 4:'''
+
 
+
Holding \( p \) fixed and derive by \( q \):
+
 
+
$$ P_q |_p = - \Phi_{Qq} - \Phi_{QQ} Q_q |_p \Rightarrow P_q |_p = - \Phi_{Qq} + \frac{\Phi_{QQ}\Phi_{qq}}{\Phi_{qQ}} $$
+
 
+
using equation 2.
+
 
+
We are now prepared to calculate:  
+
 
+
$$ \{ Q, P \} = Q_qP_p - Q_pP_q = \frac{\Phi_{QQ}}{\Phi_{qQ}}  \frac{\Phi_{qq}}{\Phi_{qQ}} - \frac{1}{\Phi_{q Q}}  \left( - \Phi_{Qq} + \frac{\Phi_{QQ}\Phi_{qq}}{\Phi_{qQ}} \right) = 1 $$ by just cancelling out the terms. \( \square \)
+
 
+
( ''My first idea would have been:''
+
 
+
By the script:
+
 
+
$$ \{ Q, P \} = \langle \nabla Q , J \nabla P \rangle_{\mathbb{R}^{2n}} = 1 $$
+
 
+
with \( \nabla = \left( \frac{\partial}{\partial q_1}, ..., \frac{\partial}{\partial p_n} \right) \) and
+
 
+
$$ J = \begin{bmatrix}
+
0 & I_n \\
+
-I_n & 0 \\
+
\end{bmatrix} $$
+
 
+
and this should result in the things we want to show (by some symplectic argument or so...).)
+
 
+
''But it might be better to just show this (the property is surely right, found it in a classical mechanics textbook):''
+
 
+
 
+
\( \vdash : \{ f, g \}_{p, q} = \{ P, Q \} \{ f, g \}_{P, Q} \)
+
 
+
''Proof''
+
 
+
With simplified notation:
+
 
+
\( \{ f, g \}_{p, q} = \sum_k \left( \frac{\partial f}{\partial q_k} \frac{\partial g}{\partial p_k} - \frac{\partial f}{\partial p_k} \frac{\partial g}{\partial q_k} \right) = \sum_k \left( \frac{\partial f}{\partial Q} \frac{\partial Q}{\partial q_k} + \frac{\partial f}{\partial P} \frac{\partial P}{\partial q_k} \right) \left( \frac{\partial g}{\partial Q} \frac{\partial Q}{\partial p_k} + \frac{\partial g}{\partial P} \frac{\partial P}{\partial p_k} \right) - \sum_k \left( \frac{\partial f}{\partial Q} \frac{\partial Q}{\partial p_k} + \frac{\partial f}{\partial P} \frac{\partial P}{\partial p_k} \right) \) \(\left( \frac{\partial g}{\partial Q} \frac{\partial Q}{\partial q_k} + \frac{\partial g}{\partial P} \frac{\partial P}{\partial q_k} \right) \)
+
 
+
\( = \sum_k \left(  \frac{\partial Q}{\partial q_k} \frac{\partial P}{\partial p_k} -  \frac{\partial Q}{\partial p_k} \frac{\partial P}{\partial q_k} \right) \left( \frac{\partial f}{\partial Q} \frac{\partial g}{\partial P} - \frac{\partial f}{\partial P} \frac{\partial g}{\partial Q} \right) = \{ Q, P \}_{q, p} \cdot \{ f, g \}_{Q, P} \) \( \square \)
+
 
+
 
+
 
+
Now compute: \( \{ Q, P \}_{q, p} \cdot \{ Q_i, Q_j \}_{Q, P} = \{ Q, P \}_{q, p} \cdot \{ P_i, P_j \}_{Q, P} = 0 \) which follows from the properties of the Poisson-bracket.
+
 
+
Furthermore: \( \{ Q, P \}_{q, p} \cdot \{ Q_i, P_j \}_{Q, P} = \{ Q, P \}_{q, p} \cdot \delta_{i j} = \delta_{i j} \)
+
 
+
which solves the problem.
+

Latest revision as of 14:13, 3 August 2015

Niklaus Messerli nik@student.ethz.ch

Article Check
Problem 1 1
Problem 2 a-c):1, d):0
Problem 3 d) Wieso müssen wir zeigen dass "\(\{L_g: g \in G\}\) is a group under composition"? Wir wissen doch schon, dass \(L: G \rightarrow \mathrm{Sym}G\) und dass \(G\) und \(\mathrm{Sym}G\) Gruppen sind.
Problem 4 1
Problem 5 1
Problem 6 1
Problem 7 1
Problem 8 2
Problem 9 1
Problem 10 a by craven seems legit
Problem 11 1
Problem 12 bitch please
Problem 13
Problem 14
Problem 15 a): 1